跳到主要內容

發表文章

目前顯示的是 5月, 2011的文章

[繪圖] 塗鴉

by Chung-Han Hsieh 工具:蠟筆

[數學分析] 逐點收斂與均勻收斂(3) - Differentiation property

如果我們手邊有一個 均勻收斂的 函數 sequence $\{f_n \}$ 且假設此數列可微,我們想知道均勻收斂是否能給我們一些關於此函數sequence 微分 $\{ f_n' \}$ 的一些關聯? 首先看個例子: Example: 令 \[ f_n(x) := \frac{\sin nx}{\sqrt{n}}, \;\; (x \in \mathbb{R}, \;n=1,2,3,...) \]試回答下列問題: 1. 函數是否逐點收斂(converges pointwise)? 2. 是否均勻收斂(converges uniformly)? 3. 此函數sequence 導數 $f_n'(x)$ 為何? 4. 此函數sequence 的導數  $\{ f_n'\}$是否逐點收斂? 5. 此函數sequence 的導數  $\{ f_n'\}$是否均勻收斂? Solution 1. 首先檢驗是否逐點收斂 給定 $x \in \mathbb{R}$,我們可知道當 $n \rightarrow \infty$ 函數 sequence $\{f_n \}$為 \[\mathop {\lim }\limits_{n \to \infty } {f_n}\left( x \right) = \mathop {\lim }\limits_{n \to \infty } \frac{{\sin nx}}{{\sqrt n }} = 0 \]亦即此 $\{f_n \}$ converges pointwise 到 $0$ 2. 現在我們檢驗其是否為均勻收斂,由均勻收斂的 sup-norm 定義,我們可檢驗其 sup-norm 看是否收斂到 $0$;亦即檢驗 \[\left\| {{f_n} - f} \right\| = \mathop {\sup_{x \in \mathbb{R}} } \left| {\frac{{\sin nx}}{{\sqrt n }} - 0} \right| = \mathop {\sup_{x \in \mathbb{R}} } \left| {\frac{{\sin nx}}{{\sqrt n }}} \right| \le \frac{1}{{\sqrt n }} \]

[數學分析] 逐點收斂與均勻收斂(2) - Series version

令 $X$ 為 metric space。現在考慮 一組 函數 sequence $\{f_n \}$ 定義在集合 $E \subset X$,我們稱 $\{f_n \}$為 uniform convergence 若下列任一條件成立 1. (Definition) 對任意 $\varepsilon >0$, 存在 $N >0$ 使得 $n > N$ 對所有的 $x \in E$ \[|f_n(x) - f(x)| < \varepsilon \]2. (Cauchy criterion) 若對任意 $\varepsilon >0$, 存在 $N >0$ 使得 對任意 $x \in E$, 我們有 \[  n,m > N \Rightarrow |f_n(x) - f_m(x)| < \varepsilon \]3. (Sup-norm version) 若 \[ \lim_{n \rightarrow \infty} \sup_{x \in E} |f_n(x) - f(x)| =\lim_{n\rightarrow \infty}||f_n - f|| =0 \] 那麼現在我們看看若是一個級數而言,我們亦可討論此級數 是否 uniform convergence。故我們先給定級數收斂的定義 ==================== Definition: (Convergence of Series of numbers ) 令級數 $ \sum_{n=1}^{\infty}a_n$, 其中 $a_n \in \mathbb{R}$ ,我們稱此級數收斂若下列條件成立: 對任意 $\varepsilon >0$,存在 $N>0$ 使得 對任意 $m > N$ \[\left| {\sum\limits_{n = 1}^\infty  {{a_n}}  - \sum\limits_{n = 1}^m {{a_n}} } \right| < \varepsilon \]亦即所謂的級數的 partial sum 收斂。 ==================== 那麼對於一組函數級數的收斂該怎麼定義呢? ==================== Definit

[衍生商品] 希臘值與動態避險 (2) - Gamma and Gamma Neutrality

延續上篇 [衍生商品] 希臘值與動態避險 (1)-Delta Hedging Example ,這次要介紹 希臘值 Gamma: $\Gamma$,此參數定義為 \[ \Gamma := \frac{\partial^2 f}{\partial S^2} \] 亦即為標的資產價格 $S$ 的二次偏導數。 注意到之前我們定義過 $\Delta := \frac{\partial f}{\partial S}$,故 $\Gamma$ 可視為選擇權 $\Delta $ 的變化 與 標的資產價格 $S$ 變化的比率。 Comment 1. 當 $\Gamma $ 很小的時候,表示 $\Delta$ 變化緩慢 (stable $\Delta$) (亦即對標的資產價格變動不敏感),故此時對於 $\Delta$-Hedging 所需的 Rebalance 不需太過頻繁。但是若 $\Gamma$ 很大的時候,表示 $\Delta$ 變化劇烈,亦即對標的資產價格變動非常敏感,故此時 $\Delta$-Hedging 需要頻繁的做 Rebalance 來確保 Delta-Neutral ($\Delta =0$)。 2. 如果考慮的是一個 選擇權交易組合的 $\Gamma$,則其定義為 \[ \Gamma := \frac{\partial^2 \Pi}{\partial S^2} \] 其中 $\Pi$ 為選擇權交易投資組合的價格。 假定且我們假設 標的資產的波動度為 Constant,則投資組合的價格為資產價格 $S$ 與 時間 $t$ 的函數,亦即我們可對 $\Delta \Pi$ 做泰勒展開求資產價格的變化 \[ \small{\Delta \Pi  = \underbrace {\frac{{\partial \Pi }}{{\partial S}}}_\Delta \Delta S + \underbrace {\frac{{\partial \Pi }}{{\partial t}}}_\Theta \Delta t + \frac{1}{2}\underbrace {\frac{{{\partial ^2}\Pi }}{{\partial {S^2}}}}_\Gamma \Delta {S^2} + \frac{1}{2}\frac{

[衍生商品] 希臘值與動態避險 (1)-Delta Hedging Example

回憶前篇  [衍生商品] 希臘值與動態避險 (0) - Delta and Delta Neutral  ,這次要介紹如何利用 $\Delta$ 進行動態避險。 回憶 $\Delta$ 定義如下: \[ \Delta := \frac{\partial f}{ \partial S} \]亦即表示為 選擇權價格 $f$ 對 股價 $S$ 的變化率。(由於其為一階導數,故為斜率) 現在來看個例子: Example 1 : (Delta Hedging) 如果 $\Delta = 0.6$ 則表示當股價 些微變化 的時候,對應的選擇權價格變化大約是股價變化值 的 $60 \%$。 下圖顯示了一組 $\Delta$ 值在某時刻的例子: 考慮上圖,假設股價為 $\$ 100 $,Call option 價格為 $ c= \$ 10$,現在考慮某金融機構的交易員賣出了 $20$ 份 Call option (一份選擇權對應其持有者可以有權購買 $100$ 股,亦即 $20$ 份call option 共 $x= 20 \times 100 = 2000$ 股)。此時如果不進行避險,則當股價上升時,該交易員會暴露風險之中: 簡單的說,現在有兩個人物: 賣出  call option 的交易員 跟交易員  購買  call option 的客戶 此時客戶的 $\Delta_{Customer} =0.6$ (因為購入call option,當股價上升對顧客有利,此時 $\Delta >0$) 而交易員的 $\Delta_{trader} = -0.6$, (由於交易員是 "賣出" 選擇權,故當股票價格上升,則選擇權會被執行,此情形時將對交易員產生風險。故此 $\Delta$ 對 交易員而言是負值) 現在,站在交易員的觀點,如果不進行避險,則交易員本身的潛在損失為 \[ -0.6 \times 2000 =-1200 \ \text{shares} \] 我們必須消除賣出 Call option所帶來的 風險,此時交易員可進行 $\Delta$-Hedging  來補足缺少的 $1200$ 股 股票。: 由於交易員是 "賣出" Call option ,故避險方法便是進行反

[衍生商品] 淺談選擇權 (2) - Put-Call Parity

這次要介紹的是選擇權訂價一個重要的關係:買權賣權等價關係 (Put-Call Parity): 想法: 利用 Option 建構一個 合成的 Forward contract ,再來比較其 payoff 現在考慮 當前股價為 $S_0$ 的 無配發股息的股票 (Non-dividend paying stock),且 $C, P$ 為對應的 Call option 與 Put option 的價格;現在我們透過 Buying a call + Short a put 可以得到 Synthetic forward 如下圖 Payoff (點圖放大) 注意到上圖右方合成之後的 Payoff 圖等價 Long a forward ,故我們稱此利用 Option 所合成出來的 Forward 為 Synthetic forward。 另外,如果我們觀察對於 Synthetic Forward 而言,我們是透過 Buying a call + short a put 達成,故其當前的 Payoff  (Payoff Today) 可寫為 $-C + P$ (買一份 call $ = -C$,賣出一份 put $=+p$);且 Payoff at Expriation date 為 $S_T - K$ 另外對於標準 Forward 而言,其 Payoff Today $=0$ (由於 Forward 並不需支付premium,故 Payoff today =0);而 Pay off at Expriation date 為 $S_T -F_0$ 因為我們的 Synthetic Forward 是用來 mimic 標準的 Forward ,故此兩者之 Payoff 必須等價,我們將所有的 payoff 都折現到 Today 來比較: Payoff Today : \[ -C + P + PV(S_T - K) = 0 + PV(S_T - F_0) \\ \Rightarrow C-P =  PV( F_0 - K) \] 又由於對 Forward 而言 $PV(F_0) := S_0$,故我們得到 \[ C-P =  S_0 - PV( K) \] 上式稱為  歐式選擇權 無 支付股息 的 Put-Call Parity。

[衍生商品] 淺談選擇權 (1) - Some Properties of Option

延續上篇 [衍生商品] 淺談選擇權 (0) - Moneyness and profit/payoff of Option  ,我們目標是要找出合理的定價。但目前對上述並無頭緒,只知道 選擇權價值 = 內在價值與時間價值。 \[ \text{Option Value $=$ Intrinsic Value $+$ Time Value} \] 然後 由報價中,我們可以發現有一些參數似乎會影響我們對選擇權的定價。 現在我們總結需要的參數: $K$: 執行價格 (Strike Price) $S_0$: 當前標的資產價格 (這邊我們以當前股價表示) (Current Stock Price) $\sigma$: 股價波動度 (Volatility) $T$ : 到期時間 (Expiration time) $r$ : 無風險利率 (risk-free interest rate) $D$: 股息 (Dividend) Style: 美式選擇權 或者 歐式選擇權。 接著我們討論當上述參數變動的時候,會對選擇權價格造成甚麼影響? Varying  Strike Price $K$: 現在考慮兩個不同的執行價格 $K_1, K_2$ 且 $K_1 < K_2$,則我們知道對於 Call option 而言,越低的執行價格代表越此 Call option 獲利機會相對較大,故Call option 售價在較低的 執行價格 應越高 \[ C(K_1) > C(K_2) \] 對 Put Option 而言,情況則相反 \[ P(K_1) < P(K_2) \] 那麼現在如果我們考慮兩選擇權除了 Strike price 以外其餘參數皆相同,則我們有如下重要結果: \[\left\{ \begin{array}{l} {K_2} - {K_1} \ge C({K_1}) - C({K_2}) \ge 0\\ {K_2} - {K_1} \ge P({K_2}) - P({K_1}) \ge 0 \end{array} \right.\] Varying   Expiration Time $T$: 1. 對於美式選擇權而言,越長的 $T$ 表示有越多機會可以 執行,故 $T$ 增大 $\Rig

[衍生商品] 淺談選擇權 (0) - Moneyness and profit/payoff of Option

選擇權 (Option) 定義: 為一個非綁定合約,給予 持有者 在合約上約定的 (未來)到期日期 (Expriation date),依照合約上約定的 執行價格 (Strike price) ,買入或者賣出標的資產 (EX: 股價、指數、外匯利率) 的權利。 Comment 1. 上述 非綁定合約,意指 持有者不一定要履行選擇權。亦可選擇放棄履行 2. 選擇權 與 期貨 (futures) 最大差別在於 選擇權買方需先支付權利金 (premium) 給賣方。但期貨無須先支付權利金。 選擇權依照分類有 兩種 : 1. 買權 (Call option):  在指定到期日期,持有者有權利以 Strike price 購買 標的資產 下圖為購買一組買權: Long (=Buy) a Call Option 在到期日的 Payoff 與 Profits (點圖放大) 上圖中 $C$ 表示 Call option 價格,而 $FV(C)$ 則為 到期時刻 Call option 的 future value,$K$ 為執行價格, $S_T$ 為到期日時的股價。其 Payoff 與 Profit 用數學表示可寫為: \[\left\{ \begin{array}{l} {\rm{Payoff}}{{\rm{}}_{Call}}:\max \left\{ {{S_T} - K,0} \right\}\\ {\rm{Profit}}{{\rm{}}_{Call}}:\max \left\{ {{S_T} - K,0} \right\} - FV\left( C \right) \end{array} \right.\] 2.  賣權 (Put option):  在指定到期日期,持有者有權利以 Strike price 賣出 標的資產 下圖為 購買一組賣權 Long (=Buy) a Put Option 在到期日的 Payoff 與 Profits (點圖放大) 上圖中 $P$ 表示 Put option 價格,而 $FV(P)$ 則為 到期時刻 Put option 的 future value,$K$ 為執行價格, $S_T$ 為到期日時的股價。其 Payoff 與 Profit 用數學表示可寫為: \[\lef

[最佳控制] 離散時間 穩態 LQR 控制問題 (1)

延續前篇,這次要介紹的是 Discrete Time Linear Quadratic Regulator in Infinite Horizon 或稱 Steady State LQR。 ================ LQR Problem (Infinite Horizon LQR): 考慮離散狀態方程: \[ x(k+1) = A x(k) + B u(k) \]其中 $x(k) \in \mathbb{R}^n, A\in \mathbb{R}^{n \times n}, B \in \mathbb{R}^{n \times m}, u(k) \in \mathbb{R}^{m \times 1}$且 $(A,B)$ controllable。 定義 Performance index: \[ J(u) = \displaystyle \sum_{k=0}^{\infty} x^T(k+1) Q x(k+1) + u^T(k) R u(k) \] 其中 $Q, R$ 必須滿足 $Q^T = Q, Q \succ 0$, $R^T = R, R \succ 0$。 (亦即 $Q, R$ 必須為  對稱 + 正定  矩陣) 試求出一組最佳控制力序列 $u^*$ 使得成本函數 $J(u)$ 最小。 ================ Comment: 讀者須注意到 Infinite Horizon 的 LQR問題要求計算 Performance index 為無窮級數和,此解必須保證收斂。以下定理告訴我們何時 此 Performance index 收斂 Lemma 考慮離散系統 $x(k+1) = A x(k) + B u(k)$,若 $(A,B)$ 可控制,且選 $Q, R >0$ 為正定矩陣,則上述 infinite horizon LQR 問題保證 閉迴路系統 狀態收斂到 $0$ 且 cost 為有界。 Proof: omitted. (see J. B. Rawlings and D. Q. Mayne, "Model Predictive Control: Theory and Design, p. 24", 2009) 現在我們可以開始求解 Infinite Horizon

[最佳控制] 離散時間 LQR- Finite Time Horizon

這次要介紹 控制理論中一個重要的結果:  Discrete Time Linear Quadratic Regulator (LQR) in Finite Time Horizon, 中文翻譯為 離散時間線性二次調節器,我們這邊會針對此問題利用 Dynamic Programming 的方法來逐步求解 ================ LQR Problem (Finite Horizon): 考慮狀態方程: \[ x(k+1) = A x(k) + B u(k) \]其中 $x(k) \in \mathbb{R}^n, A\in \mathbb{R}^{n \times n}, B \in \mathbb{R}^{n \times m}, u(k) \in \mathbb{R}^{m \times 1}$ 且 考慮 Performance index: \[ J(u) = \displaystyle \sum_{k=0}^{N-1} x^T(k+1) Q x(k+1) + u^T(k) R u(k) \] 其中 $Q, R$ 必須滿足 $Q^T = Q, Q \succ 0$, $R^T = R, R \succ 0$。 (亦即 $Q, R$ 必須為 對稱 + 正定  矩陣) 試求出一組最佳控制力序列 $u(N-1), u(N-2),... u(0)$ 使得成本函數 $J(u)$ 最小。 ================ Comment: 1. LQR 顧名思義是其具有系統狀態方程為線性 $x(k+1) = Ax(k)+Bu(k)$ 與  Performance Index 中的項都為二次式。 \[ J(u) = \displaystyle \sum_{k=0}^{N-1} x^T(k+1) Q x(k+1) + u^T(k) R u(k) \] 2. 上述對於矩陣 $Q, R$ 的對稱與正定假設是必須的 (之後需求在求解最佳控制力序列的時候需要求解反矩陣,故需要這些性質。) 3. 上述 LQR in Finite Horizon的問題所求得的最佳控制力序列為 Time Varying 。(此性質會在下面求解的時候再度強調。) 4. 若 Performance index 考慮 $N \rightarrow \